0% found this document useful (0 votes)
60 views

T9SOL

The document is a problem set from a university course on metric spaces. It contains 10 problems and their solutions. Problem 1 proves the nested interval property using the least upper bound axiom. Problem 2 shows that the diameters of a nested sequence of intervals approaches 0 as n approaches infinity, and their intersection is empty. Problems 3-5 provide other examples of nested sequences and their intersections. Problems 6-10 prove various properties about Cauchy sequences, equivalence of sequences, and completeness of metric spaces.

Uploaded by

Ajoy Sharma
Copyright
© © All Rights Reserved
We take content rights seriously. If you suspect this is your content, claim it here.
Available Formats
Download as PDF, TXT or read online on Scribd
0% found this document useful (0 votes)
60 views

T9SOL

The document is a problem set from a university course on metric spaces. It contains 10 problems and their solutions. Problem 1 proves the nested interval property using the least upper bound axiom. Problem 2 shows that the diameters of a nested sequence of intervals approaches 0 as n approaches infinity, and their intersection is empty. Problems 3-5 provide other examples of nested sequences and their intersections. Problems 6-10 prove various properties about Cauchy sequences, equivalence of sequences, and completeness of metric spaces.

Uploaded by

Ajoy Sharma
Copyright
© © All Rights Reserved
We take content rights seriously. If you suspect this is your content, claim it here.
Available Formats
Download as PDF, TXT or read online on Scribd
You are on page 1/ 6

The University of Sydney MATH 3901

Metric Spaces

2004

Tutorial 9 PROBLEM SET 9 1. Use the least upper bound axiom to prove the nested interval property: Let (In ), where In = [an , bn ], be a nested sequence of closed (and bounded) intervals in R. Prove that there exists at least one point common to every interval. Moreover, if limn |In | = 0, where |In | denotes the length of In , then prove that there exist exactly one point common to every interval.

Solution. Since In In+1 , it follows that a1 a2 . . . and . . . b2 b1 .

We claim that, for all m, n = 1, 2, . . . , am < bn . In fact, if m > n, then am < bm bn and if m n, then am an < bn . Thus each bn is an upper bound for the set A = {a1 , a2 , . . . } of left end points. By the least upper bound axiom of R, sup A exists; say, p = sup A. Since each bn is an upper bound for A and p is the least upper bound, it follows that p bn , for all n = 1, 2, . . . . Next, since p is an upper bound for A, we see that an p, for all n = 1, 2, . . . . Hence an p bn , for all n = 1, 2, . . . . Since each In is an interval, we conclude that p In = [an bn ], for all n. Hence p is common to every interval. Suppose that limn |In | = limn (bn an ) = 0. Then for any > 0, there exists a positive integer K such that whenever n > K , bn an < . Suppose that p1 and p2 belong to every interval. If p1 = p2 , then |p1 p2 | = > 0. Since limn |In | = limn (bn an ) = 0 , there exists an interval IK = [aK , bK ] such that the length of IK is less that the distance |p1 p2 | = between p1 and p2 . Thus p1 and p2 cannot both belong to IK , a contradiction. Hence p1 = p2 , that is, there exists exactly one point common to every interval.

2 2. In R, let An = (0, n1 ), for n = 1, 2, . . . . Show that d(An ) 0 as n ,

and that
n=1

An = .

Solution. Note that (An ) is a nested sequence. We have for n = 1, 2, . . . that d(An ) = sup d(x, y ) =
x, y An

1 , n

and so d(An ) 0 as n . If there is a p common to every interval An , then 0 < p < 1/n, for all n. Since 0 < p, there is a positive integer K such that 0 < 1/K < p which implies that

p / AK , a contradiction. Hence
n=1

An = .

3.

In R, let An = [n, ), for n = 1, 2, . . . . Show that


n=1

An = .

Solution. If p is a point common to every interval An , then n < p < , for all n = 1, 2, . . . . But p is not greater than p + 1 so that p / Ap+1 , a contradiction.

Hence
n=1

An = .

4.

Give an example of a nested sequence of open intervals in R with one point common to every interval.

Solution. In R, let A1 = (1, 1), A2 = (1/2, 1/2), . . . , Ak = (1/k, 1/k ), . . . . Then (An ) is a nested sequence of open intervals in R. We have shown in

Question 3/Problem Set 1 that


k=1

Ak = {0}. Hence 0 is a point common to

every interval. 5. Let X = (0, 1) with the Euclidean metric d. Give an example of a nested sequence (An ) of non-empty closed sets in X with d(An ) 0 as n , but

An = .
n=1

Solution. Note that (X, d) is not a complete metric space. Let An = (0, n1 ], for n = 2, 3, . . . . Then (An ) is a nested sequence of non-empty closed sets in X . A similar arguments as in Question 2 shows that d(An ) 0 as n , and

An = .
n=1

3 6. Let (X, d) be a metric space. Prove that (X, d) is complete if and only if every nested sequence (An ) of nonempty closed sets in X , with diameters d(An )

tending to zero, has exactly one point in the intersection


n=1

An .

Solution. Suppose that X is complete and let (An ) be a nested sequence of nonempty closed sets in X such that diameters d(An ) 0 as n . Since each An is non-empty, we can choose a sequence (xn ) such that xn An for each n. Then (xn ) is a Cauchy sequence. In fact, let > 0. Then since lim d(An ) = 0,
n

there exists a positive integer K such that d(AK ) < . Since (An ) is a nested sequence, it follows that for any n, m > K , An AK and Am AK so that xn , xm AK which implies that d(xn , xm ) < . Hence (xn ) is a Cauchy sequence. Since X is complete (xn ) converges to a point p in X . We claim

that p
n=1

An . Suppose not, that is, suppose there is K such that p / AK .

Then p X \ AK . Since AK is closed, X \ AK is open and so there is an open ball B (p; ) such that B (p; ) AK = . But n > K implies xn AK which

implies xn / B (p; ). This is impossible, since xn p. Hence p


n=1

An .

Suppose that p1 and p2 are common to every An . If p1 = p2 , then d(p1 , p2 ) = > 0. Let > 0 be such that < and choose K such that d(AK ) < . Since p1 , p2 AK , it follows that d(p1 , p2 ) < < , a contradiction. Hence p1 = p2 . Suppose that the condition holds and let (xn ) be a Cauchy sequence in X . Set A1 = {x1 , x2 , . . . }, A2 = {x2 , x3 , . . . }, . . . . Let > 0. Since (xn ) is a Cauchy sequence, there exists K > 0 such that for all m, n > K , d(xn , xm ) < . Thus, for n > K , d(An ) < and so lim d(An ) = 0. Since d(A) = d(A), where
n

A is the closure of A, it follows that A1 A2 . . . is a (nested) sequence of nonempty closed sets whose diameters tend to zero. An = ; say p An . We claim that the Cauchy Hence by the hypothesis,
n n n

sequence (xn ) converges to p. Let > 0. Since lim d(An ) = 0, there is N > 0 such that d(AN ) < ,. Then for n > N , we see that an , p AN so that d(xn , p) < . Hence (xn ) converges to p. This completes the proof.

4 7. Prove that if X is complete and Y is isometric with X , then Y is also complete.

Solution. Suppose that X is complete and Y is isometric with X . Let f : X Y be an isometry. Let (yn ) be a Cauchy sequence in Y and let xn X be such that f (xn ) = yn , for all n. Since f is an isometry, it follows that d(xn , xm ) = d f (xn ), f (xm ) = d(yn , ym ) so that (xn ) is a Cauchy sequence in X , since (yn ) is. Since X is complete, the Cauchy sequence (xn ) converges to a limit x in X ; that is, lim d(xn , x) = 0.
n

Let y = f (x). Then y Y and d(yn , y ) = d f (xn ), f (x) = d(xn , x) since f is an isometry. Thus lim d(yn , y ) = 0; that is (yn ) converges to y in n Y . Hence Y is complete. 8. Let X = (X, d) be a metric space and CS (X ) the collection of all Cauchy sequences in X . For (xn ) and (yn ) in CS (X ), dene (xn ) (yn ) if and only if
n

lim d(xn , yn ) = 0.

Show that is an equivalence relation CS (X ). Solution. Since d(xn , xn ) = 0, for all n, we see that for all (xn ) CS (X ), (xn ) (xn ) . Next, since d(x, y ) = d(y, x) for all x, y X , it follows that (xn ) (yn ) = (yn ) (xn )

for all (xn ), (yn ) CS (X ). If (xn ) (yn ) and (yn ) (zn ), for (xn ), (yn ), (zn ) CS (X ), then
n

lim d(xn , yn ) = 0

and
n

lim d(yn , zn ) = 0.

Since d(xn , zn ) d(xn , yn ) + d(yn , zn ), it follows that


n

lim d(xn , zn ) lim d(xn , yn ) + lim d(yn , zn ) = 0 + 0 = 0.


n n

Therefore (xn ) (zn ) . Hence is an equivalence relation on CS (X ).

5 9. Prove that two convergent sequences in a metric space (X, d) are equivalent if and only if they have the same limit.

Solution. Let (xn ) and (yn ) be two convergent sequences with limits x and y, respectively; that is, lim d(xn , x) = 0 and lim d(yn , y ) = 0.
n n

Suppose that (xn ) and (yn ) are equivalent. Then


n

lim d(xn , yn ) = 0.

Now d(x, y ) d(x, xn ) + d(xn , yn ) + d(yn , y ) and as n , we see that the right hand side tends to 0. Thus d(x, y ) 0 so that d(x, y ) = 0 and hencex = y. Now suppose that x = y . Since d(xn , yn ) d(xn , x) + d(x, y ) + d(y, yn ) = d(xn , x) + d(yn , y ), it follows that
n

lim d(xn , yn ) = 0,

and so (xn ) and (yn ) are equivalent. 10. Show that (xn ) (yn ) if and only if they are both subsequences of some Cauchy sequence (zn ). Solution. Suppose that (xn ) (yn ); that is, lim d(xn , yn ) = 0. Dene a sequence (zn ) n by xn/2 , if n is even, zn = y(n+1)/2 , if n is odd. That is, (zn ) = (y1 , x1 , y2 , x2 , . . . ). We claim that (zn ) is a Cauchy sequence. Let > 0. Then there exist positive integers K1 , K2 , K3 , such that m, n > K1 = d(xm , xn ) < /2; m, n > K2 = d(ym , yn ) < /2; n > K3 = d(xn , yn ) < /2. Set K = max(K1 , K2 , K3 ). Then we will show that m, n > 2K = d(zm , zn ) < .

6 Note that, if m > 2K , then m/2 > K1 , K2 and (m + 1)/2 > K2 , K3 . Thus we see that m, n even zm = xm/2 , zn = xn/2 d(zm , zn ) < /2 < ; m, n odd zm = y(m+1)/2 , zn = y(n+1)/2 d(zm , zn ) < /2 < ; m even, n odd zm = xm/2 , zn = y(n+1)/2 d(zm , zn ) < d(xm/2 , ym/2 ) + d(ym/2 , y(n+1)/2 ) < /2 + /2 = ; and so (zn ) is a Cauchy sequence; that is, (xn ) and (yn ) are both subsequences of some Cauchy sequence (zn ). Conversely, suppose that (xn ) and (yn ) are both subsequences of some Cauchy sequence (zn ); say, (xn ) = (zjn ) and (yn ) = (zkn ). Then
n

lim d(xn , yn ) = lim d(zjn , zkn ) = 0,


n

since (zn ) is Cauchy and n implies jn , kn . Hence (xn ) (yn ).

You might also like